25-я Балканская математическая олимпиада
Охрид, Македония, 2008 год


Существует ли последовательность положительных действительных чисел $ a_1,a_2,\ldots$ удовлетворяющая следующим двум условиям одновременно:
i) $ a_1+a_2+\ldots+a_n\le n^2$, для каждого натурального $n$;
ii) $ \dfrac1{a_1}+\dfrac1{a_2}+\ldots+\dfrac1{a_n}\le2008$, для каждого натурального $n$?
посмотреть в олимпиаде

Комментарий/решение:

  6
2023-05-28 03:05:27.0 #

Допустим такая последовательность существует, тогда

\[ a_{k+1} + \ldots + a_{2k} < a_{1} + \ldots + a_{2k} \le 4k^2. \]

Далее по неравенству Коши

\[ \frac{1}{a_{k+1}} + \ldots + \frac{1}{a_{2k}} \ge \frac{k^2}{a_{k+1} + \ldots + a_{2k}} > \frac{1}{4}. \]

Следовательно

\[ \frac{1}{a_{1}} + \underbrace{\frac{1}{a_{2}}}_{> 1/4} + \underbrace{\frac{1}{a_{3}}+\frac{1}{a_{4}}}_{>1/4} + \underbrace{\frac{1}{a_{5}}+\ldots+\frac{1}{a_{8}}}_{>1/4} + \ldots > \frac 14 + \frac 14 + \frac 14 + \ldots \]

откуда ясно, что эта сумма рано или поздно станет больше $2008,$ противоречие.